Você está na página 1de 28

566 DOCUMENT ANALYSIS/Analytical Methods

DOCUMENT ANALYSIS

Contents
Analytical Methods
Document Dating
Forgery/Counterfeits
Handwriting
Ink Analysis

technique is also used for paper fiber analysis that


Analytical Methods allows the type of fibrous raw materials used for
V Aginsky, Riley, Welsh and Associates, Forensic making the paper of Q documents to be determined.
Document Examinations, Inc., East Lansing, MI, USA Scanning electron microscopy is used when a highly
Copyright # 2000 Academic Press
magnified image (down to the nanometer range) of a
micro-fragment of ink on paper or of paper itself is
doi:10.1006/rwfs.2000.0477
desired, for example, in cases when the sequence of
crossing strokes is under examination.
Introduction
Tasks often requested for the analysis of documents
Chemical Reactions
involve distinguishing inks on the same or different
documents and also identifying the source or date of a Spot chemical (color or crystal) tests are known to
particular ink or paper. Usually these types of deter- have been used for more than a hundred years for
minations are conducted with the help of analytical detecting both inorganic and organic ingredients of
methods. There are many such methods available and inks and paper. The spot or solubility tests are carried
the document examiner should know their capabil- out both in situ (on the document itself) or on a
ities and limitations. removed sample. These tests are used to differentiate
ink formulas, to presumptively identify the constitu-
ents of an ink formula, or to select a solvent suitable
for the following extraction of the ink.
Optical Examinations
Spot color and solubility tests have been used for
Practically each examination of a questioned (Q) determining the sequence of crossing strokes of dif-
document starts with simple optical methods which ferent inks and for evaluating the relative age of inks
allow observations in ultraviolet (UV), visible (natural of the same formula and on the same paper.
daylight, filtered or unfiltered artificial light) or near
infrared (IR) regions of the electromagnetic spectrum.
These methods analyze color and luminescence of ink
Spectroscopic Techniques
on paper, security fibers in paper, etc. They may also
help in viewing obliterated writings, exposing altera- Spectroscopic methods measure the absorption, emis-
tions, erasures and substitutions, and discriminating sion, or scattering of electromagnetic radiation by
between writing inks that appear to be of the same atoms or molecules of compounds. The resulting
color. spectra of the absorption, emission, or scattering of
The morphology of ink lines on paper is examined light are functions of wavelength and depend on the
with the help of optical microscopy that uses visible energy level structure of atoms or molecules. These
light for illumination, and glass lenses for magnifying spectra are useful for characterizing and identifying
and focusing. This may allow classification of inks (e.g. with infrared spectra) compounds.
being examined, discrimination between inks being X-ray fluorescence is commonly used for solids in
compared, or may, in rare occasions, individualize which secondary X-ray emission is generated by
the writing instrument through its performance char- excitation of a sample with X-rays. The technique
acteristics. In combination with spot color tests, this has found extensive applications in determining the
DOCUMENT ANALYSIS/Analytical Methods 567

elemental profile of the ink and paper of suspect cur- practically impossible; as a rule, only characterization
rency. This profile is then compared with the profile of the major functional groups of the compound can
of genuine currency to uncover inconsistencies. be accomplished. In order to produce conclusive
Energy dispersive X-ray microanalysis combined identification, either peak-to-peak correlation using
with scanning electron microscopy (SEM-EDX) is the spectrum of a known sample or a comprehensive
an important analytical method due to its ability to software library of IR spectra would be required.
examine surface morphology with high resolution Fourier transform infrared (FT-IR) spectroscopy
and depth of field, and to produce qualitative and has been used for the characterization of organic
quantitative elemental analyses of selected microareas components in many materials commonly examined
by detection of characteristic X-rays. Many writing during document analysis (ink, paper, photocopier
and printing inks contain detectable elements that can toners, correcting fluids, etc.).
be used for characterization and comparison by SEM- Diffuse reflectance infrared Fourier transform spec-
EDX. In addition some writing inks have detectable troscopy (DRIFTS) has been found to be a reliable,
trace rare organometallic compounds added which reproducible and selective technique for the classifi-
indicate the year of its production. Finally, the tech- cation and identification of photocopier toners. Com-
nique is a useful tool for the classification and differ- pared with conventional dispersive IR spectroscopy,
entiation of photocopier toners. In particular, it is the DRIFTS technique provides spectra with a signif-
useful in recognizing monocomponent process toners icantly improved signal-to-noise ratio, and therefore,
as they contain magnetic carrier materials (magnetite, it more effectively extracts data from toners that are
ferrite) which are easily detected by SEM-EDX. normally highly absorbing in the infrared due to the
SEM-EDX has also been applied to the character- large proportion of carbon black content.
ization of trace elemental profiles of pencils. The Recently, FT-IR microspectrophotometry (a micro-
SEM-EDX analysis of both plain and coated photo- scope attachment allows the infrared beam to be
copy paper has been used to provide a comparison, focused on an extremely small area) has been exten-
detect batch differences or to ensure that the contents sively used for the characterization and differentia-
of the minor inorganic components detected in the tion of writing inks and photocopier toners.
ink or toner samples cut out of the document, are not IR spectroscopy can also be used for document
the result of their contact with the paper. analysis in combination with other techniques.
Other analytical methods that also have been used Thus, ink resin can undergo pyrolysis (see below),
for determining the elemental composition of ink and followed by IR analysis of the volatile gases gener-
paper are inductively coupled plasma mass spectro- ated. In most cases, spectra of the pyrolysis products
metry (ICP-MS) and neutron activation analysis resemble those of the parent substances. Even when
(NAA). they do not, the spectra are fairly reproducible; thus
Ultraviolet and visible absorption spectroscopy is the reference spectrum of a known substance pre-
used mostly for the analysis of organic materials. It pared in the same manner can be used for comparison
measures the wavelength and intensity of absorption with the material (ink, toner, paper) analyzed.
of near-ultraviolet and visible light by a sample. Raman spectroscopy (an emission technique in
UV±visible reflectance microspectrophotometry which a laser is directed onto the sample and a very
has been applied to measuring reflectance (absor- small fraction of the scattered radiation displaced
bance) electronic spectra of ink on paper. The method from the laser wavenumber by the vibrational wave-
allows discrimination between similarly colored inks numbers of the sample, is measured) is used for the
at a considerably higher degree of certainty than it analysis of inks and photocopying toners in a manner
could be done using optical microscopy or evaluation similar to IR spectroscopy.
by the unaided eye. Microspectrofluorimetry has
been used for measuring the emission spectra of ink
Chromatographic Techniques
on paper and of some additives to paper (fluorescent
fibers, optical brighteners). Chromatography is a method used to separate, char-
Infrared spectroscopy measures the wavelength acterize and identify (e.g. with mass spectrometry) the
and intensity of the absorption of mid-infrared light components of a mixture. Since its introduction in
by a sample. As the wavelengths of IR absorption 1903 chromatography has become a separation
bands are characteristic of specific types of chemical method that is now a widely accepted and recognized
bonds, IR spectroscopy can be used to identify com- technique.
pounds. It should be stressed, however, that, if the In document analysis, chromatographic techniques
components of interest are analyzed without isolating are extensively used for the characterization, compar-
from the matrices, their chemical identification is ison, source determination and dating of ink.
568 DOCUMENT ANALYSIS/Analytical Methods

Paper chromatography ent elution of ink samples by automated multiple


development. Postchromatographic derivatization
In paper chromatography the mixture to be separated
has been used for the visualization of separated
is allowed to soak along the paper by capillary action;
chromatographic zones of colorless organic compo-
the cellulose in the paper acts as the adsorbent. The
nents of inks and paper. Scanning TLC densitometry
technique, as well as paper electrophoresis, has been
has shown a high discriminating power with regard to
used for differentiating ink samples.
inks that are indistinguishable to the eye having subtle
differences in relative proportions of their dye com-
Thin-layer chromatography (TLC)
ponents.
This is a form of liquid chromatography that is used For over the past twenty years, different approaches
for separating nonvolatile organic and inorganic using TLC have been used for determining the age of
compounds. Among other analytical techniques ink on documents. According to a so-called `static'
applied to document analysis, TLC has been most approach that deals with the analytical profiles of inks
extensively used both for discriminating inks and for that do not change with age, the examiner determines
identifying ink formulas (by comparison with a `com- the age or source of inks by using a collection of
plete' set of standards, see below). reference standards or by detecting tags, e.g. optical
A typical procedure for the TLC analysis of ink is as brighteners or other unique components specially
follows. A sample of ink dissolved in an appropriate added by the manufacturer. If the manufacturer of
solvent, is deposited as a spot (or a band) on the the ink analyzed is identified and its formula is shown
starting line of a TLC plate that consists of a station- to be unique (through a tag or unique formula known
ary phase immobilized on a glass, aluminum or plastic only by the manufacturer), the manufacturer's files
plate. The constituents of the sample can be identified are consulted to determine the initial production date
by simultaneously running standards with the un- of the ink. This allows one to establish whether a Q
known. The bottom edge of the plate is placed in a ink was available or not at the time the document was
reservoir with a solvent (mobile liquid phase); the allegedly prepared. One obvious limitation here is
solvent moves up the plate by capillary action. that only a few inks actually contain unique dating
When the solvent front reaches a certain height (e.g. tags.
the other edge of the stationary phase), the plate is Another ink dating approach measures the `dyna-
removed from the solvent reservoir. Inks are mixtures mic' characteristics of an aging ink, i.e. those that
of many components, which move up the plate at change with age. Several ink-dating techniques based
different rates due to differences in their partitioning on TLC, evaluate the age (date) of a Q entry relative
behavior between the mobile liquid phase and the to reference samples which are known dated entries
stationary phase. Most separated components of inks written by ink of the same formula as the Q entry.
are easily detected on the resulting chromatogram due These techniques primarily use TLC to identify a Q
to their own color. The other separated spots (color- ink formula. However, it should be emphasized that,
less vehicle components) can be visualized with UV in fact, unless one is certain that the formula is proven
light or by treating the plate with an appropriate to be unique (see above), the identification of the Q
chromogenic or fluorogenic reagent. ink formulation with 100% certainty is hardly possi-
Besides the characterization and differentiation of ble. The reason for this is that, on the one hand, inks
writing ink and the chemical identification of dyes of the same type and of similar color are very similar
removed from currency involved in a robbery and in their dye components (separated and detected by
exposed to an exploding dye-pack, both conventional TLC) and, on the other hand, no matter how com-
TLC and high performance thin-layer chromatogra- prehensive the collection of reference samples is, it
phy (HPTLC) have been applied to discriminating will never be complete. Hence, it follows that unless
between stamp pad and typewriter ribbon inks, print- the formula is unique, there is always a possibility that
ing inks (including those used in counterfeit cur- a true match is not in the standard ink library.
rency), photocopier toners containing dyes mixed This circumstance is of extreme importance and it
with the carbon black pigment, jet printer inks and should always be kept in mind when the examiner
papers (tinting materials, optical brighteners, sizers uses any ink dating technique that is based on the ink
and other components of paper can be separated and formula identification approach.
used to discriminate between paper samples). The
method has been used both in its normal phase
High performance liquid chromatography (HPLC)
(hydrophilic stationary phase, e.g. silica gel) and
reversed phase (hydrophobic stationary phase, e.g. HPLC is a form of liquid chromatography in which
RP-18 modified silica gel) versions, including gradi- the stationary phase is packed in a separation column.
DOCUMENT ANALYSIS/Analytical Methods 569

Components of a sample to be analyzed are separated ink and paper analysis are capillary zone electrophor-
by injecting a plug of the sample onto the column. esis (CZE) and micellar electrokinetic capillary elec-
These components pass through the column at differ- trochromatography (MECC). In CZE separation is
ent rates due to differences in their partitioning solely based on charge, but MECC enables separation
behavior between the mobile liquid phase and the of both charged and neutral or even hydrophobic
stationary phase. The presence of analytes in the molecules; it becomes possible by adding organic
column effluent is detected by measuring a change solvents and surfactants to the pH buffers.
in refractive index, UV±visible absorption at a set CE has recently been applied to the analysis of
wavelength, fluorescence after excitation with a ballpoint, roller ball, fineliner and marker pen inks,
suitable wavelength, or electrochemical response. and has shown a very high resolving power that
The separated analytes can also be identified with allows the efficient separation of both major and
the help of a mass spectrometric detector. minor components of ink dyes (including their sub-
HPLC has successfully been applied to the char- stitution derivatives and isomers) and, therefore, the
acterization and differentiation of ballpoint inks. It discrimination between inks with similar dyes from
enables a high discrimination between inks having different sources or different batches. The amount of
similar dye composition by separating and comparing ink-on-paper needed for the analysis is comparable to
their colorless components such as the ink vehicle HPLC and TLC. To detect peaks on the ink electro-
components which are reliably detected in the UV phoregram caused by the paper's constituents (optical
region of the electromagnetic spectrum. If inks that brighteners, etc.), blank paper samples of similar size
are to be compared are on different papers, samples as those taken from the inked paper should also be
taken from the papers should be analyzed by the same analyzed.
procedure used for the ink-on-paper to ensure that
sizers, optical brighteners, tinting materials and other
Gas chromatography/mass spectrometry (GC/MS)
chemicals that may present in the paper would not
interfere with the analysis. The modern HPLC pro- Gas chromatography (GC) is the most widely used
vides the examiner with a highly sensitive multi- analytical technique in forensic laboratories. The
wavelength detection system (diode array detector) technique primarily involves the use of three compo-
which will provide not only chromatographic profiles nents: an injector, a separation column (in a thermo-
of the inks being compared but also the in-situ stated oven) and a detector. After vaporization in the
recorded UV and visible spectra of each eluting heated injector, the sample is then transferred to the
peak in the chromatogram. Obviously, such a com- column through the use of a carrier gas. The indivi-
bination of chromatographic and spectral data dual sample components mix with the gas, travel
improves the ability of HPLC to discriminate between through the column and are selectively retained by
closely related inks. The ability of HPLC to discrimi- the stationary liquid phase contained within the col-
nate between similar ink samples is also enhanced by umn. Finally, a detector is utilized to produce a signal
increasing the resolving power with gradient elution. to a recording device. The resulting gas chromato-
HPLC has also been used for the analysis of non- gram is a series of peaks, each of which is character-
ballpoint pen inks, as well as printing inks (including istic of a particular substance.
those used in counterfeit currency), photocopier It has been shown that the most selective GC
toners and paper. determination of components of the complex mix-
tures can be achieved by the coupling of a micro-
mass-spectrometer (mass selective detector) and
Capillary electrophoresis (CE)
capillary GC. Mass selective detector uses the differ-
Performing electrophoresis in small-diameter capil- ence in mass-to-charge ratio of ionized molecules to
laries allows the use of high electric fields resulting in separate them from each other. Molecules have dis-
very efficient separations. Due to electroosmotic tinctive fragmentation patterns that provide struc-
flow, all sample components migrate in pH buffer tural information usually sufficient for identifying
towards the negative electrode. A small volume of substances separated by GC.
sample (a few nanoliters) is injected at the positive Thus, gas chromatography/mass spectrometry
end of the capillary and the separated components are (GC/MS) produces a mass spectral fingerprint for
detected near the negative end of the capillary. CE each sample component eluting from the column
detection is similar to detection in HPLC, and and, therefore, can allow discrimination between
includes absorbance, fluorescence, electrochemical compounds having a very similar chromatographic
and mass spectrometry. behavior (close retention indices).
Two versions of CE known to have been used for GC/MS has been used for the ink characterization,
570 DOCUMENT ANALYSIS/Document Dating

batch origin determination and ink comparison. In document dating. Part I. The static approach: determin-
the scan acquisition mode, the method allows identi- ing age independent analytical profiles. International
fication of an ink's volatile solid ingredients among Journal of Forensic Document Examiners 1:40±51.
which can be nonreacted low molecular mono- or Kirchner JG (1978) Thin Layer Chromatography, 2nd edn.
New York: Wiley Interscience.
oligomers, reagents and also proprietary additives
Lyter AH (1983) Analysis of writing inks by high
that are often contained in the resins, polymers or
performance liquid chromatography. In: Lurie IS and
other components of ink vehicles (carriers). It has Wittwer JD (eds) High Performance Liquid Chromato-
been shown that, even in old ink-on-paper, high graphy in Forensic Chemistry. New York: Marcel
boiling vehicle solvents can be detected and identified Deker.
using the selected ion monitoring (SIM) acquisition Maehly A and Stromburg L (1981) Chemical Criminalis-
mode; the detector is set to monitor ions specific to tics. New York: Springer-Verlag.
the solvents commonly used in the manufacture of Merrill RA, Bartick EG and Mazzella WD (1996) Studies
inks. of techniques for analysis of photocopy toners by IR.
Recently, the unique ability of GC/MS to efficiently Journal of Forensic Sciences 41:81±88.
separate ink volatile components and to quantify ASTM (1992) Standard guide for test methods for forensic
writing ink comparison. 1992 Annual Book of ASTM
them at down to picogram level has been successfully
Standards, Section 13, ASTM Designation: E 1422-91
used for developing ink dating techniques applicable
Philadelphia: ASTM.
to ballpoint, porous tip and roller pen inks, stamp pad Tebbett IR (1991) Chromatographic analysis of inks for
inks, inks for jet printers, and other inks containing forensic science applications. Forensic Science Review
high-boiling vehicles. 3:72±82.
Tebbett I (1992) Gas Chromatography. New York,
Pyrolysis gas chromatography
London, Toronto, Sydney, Tokyo, Singapore: Ellis
GC is capable of separating volatile organic sub- Horwood.
stances. Therefore, it is not directly applicable to Xu X, de Koeijer JA, de Moel JJM and Logtenberg H
the analysis of such nonvolatile substances as resins (1997) Ink analysis for forensic science application by
micellar electrokinetic capillary chromatography with
in inks or sizing materials in paper. However, pyro-
photodiode array detection. International Journal of
lysis of similar nonvolatile substances leads to their Forensic Document Examiners 3:240±260.
breakdown (thermal decomposition) into smaller Zeichner A, Levin N, Klein A and Novoselsky Y (1988)
compounds which are volatile enough to be analyzed Transmission and reflectance microspectrophotometry
by GC. A pyrolysis device is directly connected to the of inks. Journal of Forensic Sciences 33:1171±1184.
inlet of the gas chromatograph and the compounds
produced by pyrolysis are separated and detected by
the chromatographic system. The resulting pyrogram
is a highly specific pattern of peaks which is a
`fingerprint' of the substance analyzed.
Pyrolysis GC with mass spectrometric detection
(PyGC/MS) has been used for the characterization
Document Dating
of nonvolatile organic components in inks and photo- D C Purdy, Forensic Document Examination Services
copier toners. The technique has provided high dis- Inc., Ottawa, Canada
crimination between closely related inks and toners. Copyright # 2000 Academic Press

doi:10.1006/rwfs.2000.0483
See also: Analytical Techniques: Separation Tech-
niques; Microscopy; Spectroscopy: Basic Principles;
Presumptive Chemical Tests; Mass Spectrometry.
Document Analysis: Forgery/Counterfeits; Ink Analysis; Overview
Document Dating.
The misrepresentation of dates on documents is not a
recent challenge faced by forensic document exam-
Further Reading iners. In his book, Questioned Documents, Albert S.
Osborn provided several examples of documents
Aginsky VN (1996) Dating and characterizing writing,
which were altered or backdated to make it appear
stamp pad and jet printer inks by gas chromatography/
mass spectrometry. International Journal of Forensic as though they were written much earlier. Many
Document Examiners 2:103±115. frauds still involve document dating problems and
Brunelle RL and Reed RW (1984) Forensic Examination of forensic document examiners should diligently search
Ink and Paper. Springfield: Charles C Thomas. for any clues that suggest a document was prepared
Cantu AA (1995) A sketch of analytical methods for some time other than indicated.
DOCUMENT ANALYSIS/Document Dating 571

Various methods can be employed to backdate or The design of watermarks can also change over
fabricate documents. Such incidents can involve the time as relief areas of a dandy roll suffer damage
relatively simple process of overwriting the date on a through normal wear and tear. Detached or broken
receipt to far more complex undertakings such as wires produce slight but visible changes in the design
falsifying an entire document. Regardless of the which is transferred to the paper. Paper mills usually
method used, dating suspect documents is a very keep records when dandy roll damage occurred and
challenging problem for the document examiner when repairs were made. This information can be
and should be approached cautiously. very helpful in narrowing the period during which a
The most straightforward method for solving a watermarked paper was manufactured.
dating problem considers the types of office equip- A few paper companies have intentionally changed
ment and technologies used to produce the questioned the design of their watermarks from time to time.
document. The date on a document can be proven Such watermarks are said to contain a `date tag',
false if the instruments and materials used to produce which will often indicate the year that a sheet of
it were unavailable when it was supposedly prepared. paper was produced. For example, Southworth
A second method takes into account certain features Paper Company placed a short bar below the letters
in the contested document that vary over time. Defec- in their watermark to indicate the last digit of the year
tive letters produced by a worn typewriter or photo- in which the paper was manufactured (Fig. 1). If a
copier `trash marks' originating from dirt on the document bears a watermark that was not in exis-
platen glass are two examples of this type of evidence tence when it was allegedly dated, the genuineness of
that has dating significance. A third method involves its date must surely be challenged.
the analysis of materials that make up a suspect When using watermarks to date paper, it is strongly
document. For example, some speciality papers or recommended that the paper manufacturer be con-
writing inks contain materials added to these station- tacted to verify the time period when the noted
ery products to improve their quality. If it can be features were present.
established that these materials were introduced to the
market on a specific date, any document in which they Paper composition
are found to be present must have been prepared at a
Over the years, different fillers, surface coatings or
later time. A fourth method involves knowledge of the
chemical additives have been added during the paper
aging of several components such as ink or paper.
making process to improve the quality of the product.
The following sections describe different areas that
Other changes in the manufacturing processes have
can be examined to determine when a document was
occurred for economic or environmental reasons.
drawn up or whether its date is false. The results of
These innovations and modifications can establish
these tests do not always provide conclusive evidence
the earliest date or period a particular sheet of
of fraud. They can, however, draw attention to
paper was manufactured.
irregularities that must be reconciled before a suspect
Many North American paper manufacturers
document can be relied on as a genuine.
stopped producing acidic paper in favor of alkaline
or neutral process papers during the late 1980s and
Paper Products early 1990s. A simple pH test can indicate if a
questioned document was produced before its pur-
Watermarks ported date. This finding can be corroborated if
Conventional paper watermarks are produced during
the manufacturing process by a `dandy roll' cylinder
located at the beginning of the papermaking machine
where paper is formed into a web. The dandy roll
cylinder consists of a woven wire gauze onto which
raised designs are soldered or otherwise attached. A
watermark is created when the relief areas of the
dandy roll press into and displace paper fibers.
Paper mills usually maintain accurate records con-
cerning their watermarks. Once the paper manu-
facturer of a questioned document is known, the
company can be contacted to determine the earliest Figure 1 The short vertical bar beneath the letter `E' in this
date that a watermark design was used. Any document watermark confirms the sheet of paper was manufactured
dated earlier than this time must have been backdated. during 1966.
572 DOCUMENT ANALYSIS/Document Dating

certain chemicals that were introduced after the date


on the document are present in the paper. For exam-
ple, when mills converted their operations to an
alkaline process, many also began using calcium
carbonate (CaCO3) as a substitute for titanium diox-
ide (TiO2) in order to improve the brightness and
opacity of papers. Caution should be exercised when
interpreting such evidence and the paper manufac-
turer should be consulted to confirm when the
observed processes and materials were introduced.
Speciality papers can also contain information of
dating significance. For example, NCR (No Carbon
Required) paper first appeared in the United States
during 1954. The formula for manufacturing this
Figure 2 A notation `20EC5' on the inside flap of an envelope
product was changed several times during the 1960s represents the stock number (20), the intials of the company
and 1970s. In 1972, NCR developed a coding scheme that placed the order (EC) and the last digit in the year of
to identify the source and date of its papers. Trace manufacture (5).
amounts of various high atomic weight elements have
been added by other manufacturers as a means of
tagging their products. The dates of documents pro- year it was manufactured. The envelope manufac-
duced on speciality papers that contain tags can be turer should always be contacted to confirm the
verified by taking such information into account. accuracy of dating information.
The design and appearance of some envelopes are
unique to their manufacturers and these features may
Envelopes
well indicate when they were produced. These
Envelopes are often discarded once their contents are include, but are not limited to, the following:
removed. This is unfortunate since an envelope may
. small irregularities along the edges of the paper
contain important information about when it was
related to a damaged die stamp;
mailed and possibly when its contents were prepared.
. types of adhesives applied to the side-seams and
The following envelope areas can have dating signifi-
flap areas of the envelope;
cance: postage stamps, postage cancellation marks,
. striation patterns in adhesive layers associated with
envelope shape and printed information.
the method of application.
Postage stamps affixed to envelopes can be exam-
ined to determine if they were available when the Other areas sometimes overlooked are addresses
envelope's contents were prepared. A new postage which appear on an envelope. A particular mailing
stamp is released for sale as a `first day cover' on a or return address may not have existed when the
particular date. Postal officials or a knowledgeable document was supposed to have been sent. Postal or
stamp collector should be able to provide the precise zip codes change from time to time and these should
date a stamp was issued. Once this date is known, the always be checked to insure they existed during the
envelope and its contents must have been mailed period in question.
some time after this period.
Stamps on many envelopes bear cancellation marks
Inks and Writing Instruments
that are applied by the post office. Even if a cancella-
tion mark is not legible, the format of the mark, the One of the most challenging dating problems facing
way it was struck and the chemical composition of the document examiner is estimating when a particu-
ink can serve to establish the period when it was lar document was signed or written. If a document
applied. was supposed to have been written many years ago, it
Occasionally, logos or product codes are applied to may be possible to prove it was backdated if the type
envelopes while they are being manufactured which of pen and writing materials used were not available at
can have dating significance. The impression shown that time. Important milestone events concerning the
in Fig. 2 was found on the inside flap of an envelope development of modern writing materials are shown
manufactured by Tenison Envelope Company. This in Table 1 along with their dates of introduction.
mark represents the stock number (20), the initials of Clues as to when a document was signed can also
the company that placed the order (EC) and the be found by analyzing the questioned writing ink. A
number (5) corresponding to the last digit in the small sample of ink removed from a document can be
DOCUMENT ANALYSIS/Document Dating 573

Table 1 Significant dates of introduction in the history of writing testing. After artificially aging one sample by expos-
instruments
ing it to heat, both samples are analyzed and the test
Year Historical development
results compared. It was discovered that ink para-
meters level off after several years of natural aging.
624 Earliest reference to the quill pen This artificial aging technique is based on the hypoth-
1662 Pencils made in Nuremberg, Germany esis that induced aging will take an ink to where it
1700 Early reference to steel pens would be if it had aged naturally. If differences
1780 Steel pens made in England by Samuel Harrison
between the two tested samples are slight, the result
1857 First appearance of `copying pencils'
1945 Ballpoint pen first marketed in New York city suggests the ink entry was on the paper for some time.
1951 Felt-tip markers introduced Greater differences in solvent extraction are indica-
1955 Liquid lead pencil introduced tive of more recent ink entries.
1963 Fiber-tip pen first produced Other testing methods rely on sophisticated analy-
1968 Roller ball pen first produced
tical techniques such as gas chromatography/mass
1979 Eraser Mate erasable pen introduced by Paper Mate
spectrometry (GC/MS) to measure the concentration
of volatile components in an ink sample. This tech-
nique also requires two samples be taken from the
separated into its solid components by thin layer suspect ink entry. After exposing one to heat, both
chromatography (TLC). The result of this analysis is samples are tested and the extent to which their
a chromatogram that isolates the different dyes pre- solvent components differ provides an estimate of
sent in the ink formulation on a coated glass or plastic when the ink entry was written. This method is better
plate. Success of this method relies on the different suited for entries made within 12 months of testing.
physical and chemical properties of the ink and the The described methods are beyond all but a few
existence of a sufficiently complete set of ink refer- specialists who possess the equipment, knowledge
ence standards. and experience needed to analyze and date writing
TLC can also detect the presence of tags which have inks. Some controversy still surrounds certain ink
been added to some writing inks by their manufac- testing methods and further validation studies could
turers. During the 1970s, several US ink producers resolve these debates.
participated in an ink tagging program organized by
the Alcohol, Tobacco and Firearms (ATF) Laboratory
in the United States. This scheme urged ink manufac- Commercially Printed Documents
turers to add trace amounts of different materials Many documents subjected to forensic examinations
with distinct properties to their inks. These materials take the form of documents with letterheads, con-
would be changed annually and thereby indicate the tracts, envelopes, notary records, receipts and other
year an ink was manufactured. By 1978, approxi- types of printed stationery. Apart from typewriting,
mately 40% of writing inks produced in the United handwriting and other information they may contain,
States contained such dating tags. Although this commercial printing on documents can be used to
initiative greatly increased the ability of forensic establish whether they were produced during or after
scientists to date domestic writing inks, the continued a certain period.
growth of imported products threatened the success Minuscule printing defects such as irregular letter
of the program. Although most ink manufacturers outlines, uneven inking or small breaks in line work
withdrew from the tagging program by the early can associate a questioned document with a particular
1980s, documents purportedly written before this stationery order produced by a commercial printer.
period may contain chemical taggants that suggest Once the company that produced a printed document
they were manufactured at a much later date. is identified, more precise information about when the
Ink chemists have observed that many writing inks order was delivered and the earliest time the stock was
begin to change or age the instant they are applied to put into circulation can be determined. Access to
paper. Most people have noticed writing inks fade or samples from the order retained by the print shop
become lighter with the passage of time. In addition can also be of value when attempting to date commer-
to this obvious physical transition, investigations cially printed documents.
have shown that the chemical composition of an ink A coded mark within the body of a print job can
also changes over several months or years. These also provide important information about a print job.
effects are especially true with respect to the color, For example, the bottom of a medical form shown in
solubility and solvent volatility of the writing inks. Fig. 3 bears the notation `AC1215R0'. The printer
In 1995, Brunelle suggested a technique wherein used this number to trace the advertisement to the
two samples of a single ink entry are removed for client. Information about the order indicated when
574 DOCUMENT ANALYSIS/Document Dating

Figure 3 The alphanumeric code `AC1215R0' within this advertisement on a patient's medical form was used to establish the date
the stationery was printed.

the stationery was distributed. In this case, the adver- Table 2 Significant dates of introduction in the development of
the typewriter
tising campaign was not authorized until three years
after the patient records were supposed to have been Year Technological development
written on the printed form. This provided irrefutable
proof that the medical records had been prepared 1909 First use of bi-colored ribbon (Underwood)
long after the patient was examined by the physician. 1927 First use of carbon ribbon (Hammond-Varityper)
1944 IBM Executive proportional spaced typewriter
1956 Remington Statesman the first proportional typewriter
Typewriting by Remington
1960 First Underwood proportional spaced typewriter
The typewriting technology used to produce a ques- 1960 Underwood electric standard typewriter with duplex
tioned document is one of the first factors that should carbon and fabric ribbons
be considered when its date is at issue. During the last 1961 IBM Selectric I dual pitch single element typewriter
1963 First use of IBM Selectric polyethylene film ribbon
century, many advances have occurred in the devel- 1971 IBM Selectric II dual escapement, half backspace
opment of the modern typewriter. Some important machine
events and when they occurred are listed in Table 2. 1971 Tech III ribbon cartridge for IBM Selectric
The date a typewritten document was prepared can 1972 First daisywheel produced by Diablo Systems
be determined in other ways. One method considers 1973 IBM Correcting Selectric II with special lift-off ribbon
1975 Thermal transfer ribbon developed by IBM
the typestyle which appears on a questioned docu- 1977 First use of polyurethane ribbons (Olivetti)
ment. The shape and size of typed letters can indicate 1978 First dot matrix printer for personal computer (Epson
the make(s) and model(s) of typewriter(s) which might TX 80)
have been used to produce the typewriting. The results 1982 IBM Electronic 65 and 85 typewriters with triple pitch
of searching a large collection of typewriter specimens and right justification
1982 Brother EP-20 seven-pin thermal typewriter
can indicate that the questioned typestyle was intro- 1984 Diablo releases EPM 1 ± first thermal ribbon transfer
duced to the market on a particular date. Should the printer
typestyle's date of introduction be later than the date 1984 IBM Quietwriter with nonimpact thermal print head
on the suspect document, the questioned document 1984 Quietwriter ribbon by IBM
must certainly be regarded with suspicion.
A second method of dating typescript takes into If a typewriter is not cleaned regularly, oil, ribbon
account any typeface defects present in the questioned particles, dirt and paper fibers can accumulate within
typewritten text. Typewriters contain many moving the crevices of certain letters. When dirty typefaces
parts which gradually become worn or defective with strike the paper through the ribbon, the letters appear
use. These defective components produce misaligned filled-in rather than clear letters and numbers. These
or damaged letters that become quite apparent when imperfections will remain until the dirt is removed by
examined with a microscope. Subsequent adjustments cleaning the typefaces. Access to uncontested docu-
or repairs by a service technician can create further ments produced on the same typewriter over a period
changes to the appearance of typewriting produced by of time will reveal when changes to the appearance of
a machine. The dates when typewriter damage the typescript occurred. Fig. 4 shows how the appear-
occurred or disappeared are very significant for dating ance of typeface dirt and damage can expose a
purposes. fraudulent document.
DOCUMENT ANALYSIS/Document Dating 575

Figure 4 The questioned document could not have been typed on June 12th, 1996. Damage to the digit `9' and the filled-in body
of the `6' occurred sometime after August 6, 1996.

Typewriter single-strike and correcting ribbons can comparing the result to a database of correcting fluid
also indicate the date when documents were produced spectra. Once known, the manufacturer can be con-
on a particular typewriter. A used single-strike ribbon tacted to determine when a particular correcting fluid
will contain impressions of all the characters struck by formulation was first produced. Of course, a correct-
the machine in chronological order since the ribbon ing fluid could not have been applied to a questioned
was last changed. If the typewriter ribbon used to document before its date of introduction.
produce a questioned document is available for in-
spection, it can be examined to insure the date of a
questioned typewritten document is contempora- Photocopiers
neous with the dates of typed documents which pre- Photocopied documents that suddenly surface during
cede and follow it. If it is not, dated correspondence a litigation are often regarded with suspicion. In some
appearing immediately before and after the location cases, these documents are genuine but in other
of the question passage can serve to determine the instances, they are produced at the last moment with
approximate period when the contested document an astonishing story that they were just discovered
was typed. recently by some strange coincidence. The subject of
Correction fluids applied to conceal typing errors interest in these cases is not when the original docu-
can also help date a typewritten document. Wite-Out ment was produced but rather the date or period it
Company first introduced this product to the market was photocopied. Three facets of photocopied docu-
in 1965. In 1984, Liquid Paper introduced colored ments that have dating significance include: the copier
correcting fluid to mask corrections on different technology used, the presence of copier defects, and
colored paper stock. The presence of these materials the properties of the toner and/or paper.
on a typewritten document before their respective
introductory dates will strongly suggest a document
Copier technologies
has been backdated.
Correcting fluids are complex substances com- Just as milestone events in the development of the
posed of different resins, plasticizers, pigments, sol- typewriter are useful for dating purposes, the date of a
vents and binders. The manufacturer of a correcting copied document can be checked against the release
fluid can be identified by extracting a sample from the date of a particular office technology to insure it was
document; analyzing it by infrared spectroscopy and available when the document was allegedly produced.
576 DOCUMENT ANALYSIS/Document Dating

Different copier technologies include: (1) dual spec-


trum; (2) stabilization; (3) diffusion transfer; (4) indir-
ect electrostatic; (5) diazo; (6) dye transfer; (7) direct
electrostatic; (8) thermographic; and (9) laser. A
questioned copied document should be checked to
insure its date follows the introductory date of the
technology used to produce it, keeping in mind that
introductory dates may vary from region to region.

Examination of defects

The most straightforward means of dating photocop-


ied documents relies on defects, `trash marks' or small
flecks of toner that appear in `white' areas of a copied
document. These marks can originate from dirt,
foreign material or defects on the glass, platen cover
or photosensitive drum of the photocopier (Fig. 5).
Scratches to the glass or drum, tend to be more
permanent and will generate marks on copies pro-
duced by a machine until such time as the defective
component is removed and replaced. The temporary
nature of other defects, such as those originating from
dirt or foreign material on the glass, lid or internal
components, are temporary in that they can be re- Figure 5 The combination of photocopier `trash' marks on the
moved by cleaning the copier surfaces. Genuine questioned document (top) emerged during the period August
photocopied documents made by the same copier 3, 1995 to September 8, 1995 and could not have occurred on
July 15th, 1995 when copies of the questioned document were
that produced the questioned document provide an
allegedly prepared.
excellent means of confirming its date. Logs and
service records maintained by repair technicians are
also helpful in that they often contain photocopies
alone or both heat and pressure to fuse toner to the
produced before and after copier repairs were made.
surface of the paper. The date a given fusing process
first appeared is the earliest that a photocopy bearing
Toner analysis
this technology could have been produced.
Most photocopier toners consist of: a pigment In 1992, it was reported that indentations are
(usually carbon black); a binder which fixes the imparted to the surface of toner by damage to the
pigment to the paper (usually an organic resin such surface of a copier's fusing rollers. Fusing roller
as polystyrene); and additives used to improve the defects occur through normal wear and tear. They
properties of the toner. When any of these compo- vary with time and consequently the indentations they
nents are changed, the event can provide a useful produce in the surface of toner can be used to estimate
means of dating photocopied documents. Analysis of when a given photocopied document was produced.
photocopier toners by infrared spectroscopy and
scanning electron microscope equipment with energy
Handwriting and Signatures
dispersive spectrometry can yield information about
the chemical and physical properties of toner. A The writing of many individuals does not change
comprehensive library of toners can be used to estab- significantly for most of their adult life. However,
lish initial production dates. In some cases, the man- despite the constant and repetitive nature of devel-
ufacturer will confirm that a particular ingredient oped handwriting, practically everyone has noticed
was first used several years after the date the photo- that their signatures and handwriting do change ±
copy was supposed to be prepared. This would con- especially over long periods of time. The develop-
stitute conclusive evidence that the alleged date of the ment, progression, and eventual disappearance of
photocopy was false. handwriting features can be very helpful in solving
The process used to fuse toner to the paper can vary dating problems. Access to a quantity of specimen
from one photocopier to another. Older photocopiers material produced during a period of time can show
use cold pressure fusing wherein toner is pressed into that writers change the shape of certain letters or the
the paper surface. Newer generations use either heat form of their signatures (Fig. 6). The quantity of
DOCUMENT ANALYSIS/Document Dating 577

document was prepared after its purported date.


When preparing a postdated or backdated document,
the writer may not remember what verb tense to use.
Such inconsistencies, especially when repeated, pro-
vide a good indication that something is amiss.
When preparing business correspondence, the
typist's initials are often placed at the bottom of the
document. In fraudulent documents, the initials of a
typist who is currently employed by a company may
be used instead of the person who held the position on
the date that appears on the document.

Computer-printed Documents
Dating computer printed documents is approached in
much the same manner as dating typewritten docu-
ments. The debut of computer printer technologies
are all associated with a date of introduction. Conse-
quently, any document produced by a daisy-wheel,
dot-matrix, inkjet or laser printer cannot bear a date
Figure 6 Six signatures produced by a writer during a ten year that precedes the respective periods when these
period show some features that have a temporal significance. printers first appeared on the market.

Daisy-wheel printers
specimens required for this purpose will depend on
many factors including: (1) how rapidly the writing The daisy-wheel printer, using a similar impact tech-
changes; (2) what factor(s) influenced the changes; nology to the typewriter, bridged the gap between
and (3) the number of specimen writings prepared typewriters and later generations of computer prin-
near the period in question. Once the specimens are ters. Although very popular during the 1970s, few
arranged in chronological order, it is often possible daisy-wheel printers are still in use today. The print
to date a disputed writing within a particular time elements of these machines contain a full set of char-
period. acters positioned on the end of long spokes attached to
Rapid changes in a person's writing can result from a central hub. As the elements spin on a central shaft,
the sudden onset of a serious illness, the administra- the characters are struck at the appropriate time from
tion of therapeutic drugs or the consequence of a behind with a plunger. The action of the character
debilitating accident. Although such sudden transi- striking the paper through an inked ribbon produces a
tions can create problems for the document examiner, letter on a document.
they also provide a means of determining when a Like their typewritten counterparts, documents
questioned signature or handwriting might have been produced by daisy-wheel printers can be dated by
produced. considering irregularities in the alignment of letters
or damage to their outlines through wear and tear.
The source of other temporal defects can be traced to
Contents of a Document faulty moving components of the printer. These
Proof that a document was backdated or postdated changes provide a means for dating the work of a
can occasionally be found within its contents. These particular printer. It should be kept in mind, however,
details are often overlooked by the perpetrator as his that daisy-wheels can be easily removed, discarded
attention is focused on producing a document that and replaced by a new element. All defects associated
contains the right information. Names, addresses, with the old daisy-wheel will disappear and only those
postal codes, phone numbers, trade names, and job that relate to the printer will remain.
titles mentioned in a document might provide evi-
Dot-matrix printers
dence that it was produced at a different time.
Events are occasionally mentioned in correspon- Dot-matrix printers gained popularity during the
dence that did not occur until months or years after early 1980s. Early models had nine metal pins
the date appearing on the document. Verb tenses in arranged along a vertical axis that struck the paper
relation to events mentioned can also indicate a through an inked ribbon while the printhead moved
DOCUMENT ANALYSIS/Document Dating 579

Other factors worthy of consideration can be found Indented Writing


under the `Contents of a Document' heading of this
chapter. Indented handwritten impressions made in the sur-
face of a document can reveal important information
about whether written entries on a piece of paper
were made before or after the indented writing
Cachet Impressions occurred. Such sequence determinations are con-
firmed by subjecting the document to an ElectroStatic
The development of rubber stamps followed the dis-
Detection Apparatus (ESDA) examination.
covery of vulcanizing rubber by Charles Goodyear.
It is often possible to establish the exact date when
The first commercial production of rubber stamps
occurred in 1864. Since that time, the processes used indented handwritten impressions on a document
were produced. An ESDA examination that estab-
to manufacture stamps have undergone several
lishes the visible writing on a questioned document
improvements as the demand for better quality rub-
was made after dated indented impressions can pro-
ber stamps increased. The first pre-inked stamp,
vide an unusual but effective method for confirming
Perma Stamp was produced in 1958. These stamps
the document was backdated.
are still a popular item in stationery stores. Although
Handwritten entries in a journal, ledger, note pad
today's stamps are still referred to as `rubber stamps',
or receipt book usually produce indented impressions
most stamps are now produced from a plastic-based
photopolymer material. on underlying sheets of paper. If it is necessary to date
one of the sheets which was removed, its original
Both rubber and plastic deteriorate over time. The
location can be confirmed by matching writing on the
relief edges of a stamp can crack or break off, an ink/
document with corresponding impressions on the
dirt mixture can clog deep crevices and the relief areas
other bound papers. If the dates on adjacent pages
of a stamp can become worn through constant use.
are reliable, this simple method enables the document
These events introduce flaws that are reproduced in
examiner to place the questioned document within a
the impressions produced by a worn stamp. The
particular time frame.
approximate period when a stamp impression was
made can be determined by comparing its defects
with standards from the same stamp arranged in Guillotine Marks
chronological order.
Another method by which stamp impressions can The exposed edges of receipt books, reams of paper
be dated involves changes to the size of some stamps and stationery pads may contain marks produced by
with time. It has been found that stamps can shrink as cutters or guillotine blades used to trim these pro-
much as 1.5 mm during a four-year period. Although ducts to size. These stria, often referred to as `guillo-
this phenomenon is relatively rare, it does provide yet tine marks', do not run perpendicular to the surface of
another means of dating stamp impressions. the paper but run at an angel across the trimmed
surfaces. Their locations along the four edges of a
document can indicate where a sheet was positioned
in the original stack of paper.
Glues, Tapes and Paper Fasteners Access to several documents from the same stack of
Adhesives used to manufacture envelopes, stationery paper is needed to establish a cutting pattern against
pads and tapes occasionally undergo changes or which the contested document will be compared.
modifications to improve their properties. Such Once the location of guillotine marks on the four
changes can be used to establish the earliest date edges of the questioned sheet match the position of
that a document manufactured with a given adhesive sheets from the same lot, any dating information on
was produced. The stationery manufacturer or adhe- adjacent sheets can be used to determine when the
sive company should always be contacted to verify questioned document was written. If the questioned
the date when a particular adhesive was first used. document is not contemporaneous with information
Lift-off tape was introduced by IBM to facilitate on adjacent sheets of stationery, some plausible
the correction of typewriting errors. This innovation, explanation should be sought.
first introduced to the market by IBM on the 1st of
April 1973, removed unwanted typed characters by
Summary
overstriking letters through the lift-off tape. This
action would lift the letter from the document and Many methods can be employed to determine if a
allow the typist to correct errors with little distur- questioned document's date has been falsified. People
bance to the paper surface. who fraudulently alter or misrepresent the date of a
580 DOCUMENT ANALYSIS/Forgery/Counterfeits

document are usually careful to ensure its general Forgery/Counterfeits


appearance will meet close scrutiny. Consequently, it
is better to approach document dating problems from P W Pfefferli, Kantonspolizei, Zurich, Switzerland
a broader perspective rather than focusing attention Copyright # 2000 Academic Press
on those areas which immediately arouse suspicion.
doi:10.1006/rwfs.2000.0478
Any decision to limit the scope of forensic investiga-
tions in the interest of saving time or money should be
weighed carefully. Solutions to document dating pro- Introduction
blems are often dependent on an inspection of all
areas of a document for details confirming whether it Forensic document examination is a complex matter
was prepared on a given date. Such an approach will requiring various areas of expertise to identify docu-
insure that a thorough investigation is carried out and ment forgeries of all kinds. Although in the modern
crucial evidence will not be overlooked. office the electronically mailed document is of grow-
ing importance, the traditional (paper) document still
See also: Document Analysis: Handwriting; Analytical has its place, and thousands of different kinds are used
Methods; Forgery/Counterfeits; Ink Analysis. for documentation purposes. The threat of illegal
document forgery exists and will always exist.
The examination of questioned documents is an
Further Reading ongoing challenge to forensic sciences, to cope with
the modern technology of document manufacturing
Blackledge RD and Gernandt MN (1993) The pH pen ± a
means of comparing paper products. Journal of and the increasingly sophisticated document for-
Forensic Sciences 38:134±142. geries. The many facets of modern documents demand
Brunelle RL (1995) A sequential multiple approach to many areas of expertise from forensic specialists in
determining the relative age of writing inks. Interna- order to identify and authenticate a questioned docu-
tional Journal of Forensic Document Examiners 1:94± ment. The identification of handwriting is a discipline
98. of its own, requiring for appropriate trained experts to
Brunelle RL (1992) Ink dating ± the state of the art. Journal compare and identify handwritings. The aim of the
of Forensic Sciences 37:113±124. examination of technical typewriting and printing
Brunelle RL and Cantu AA (1987) A critical evaluation of produced by office machines is to determine typestyle
current ink dating technique. Journal of Forensic and origin. In addition to the written or printed
Sciences 32:1522±1536.
content, it is necessary to examine other document
Brunelle RL and Reed RW (1984) Forensic Examination of
Ink and Paper. Springfield: Charles C. Thomas.
components. Ink and paper examination, by nondes-
Cantu AA and Prough RS (1987) On the relative aging of tructive and analytical methods and techniques, help
ink ± the solvent extraction technique. Journal of to determine its authenticity with respect to origin.
Forensic Sciences 32:1151±1174. These areas are equally relevant in general questioned
Cantu AA (1996) A sketch of analytical method for document examination, but when documents of high
document dating Part II. The dynamic approach: value and protected by the law are fabricated for
determining age dependent analytical profiles. Interna- criminal purposes, the forensic document analysis
tional Journal of Questioned Document Examiners. may be of even higher importance and special exper-
2:192±208. (Errata in 2:570±571) tise is required. Counterfeiting and forgery of value
Gerhart J (1992) Identification of photocopiers from fusing and security documents occurs on a large scale.
roller defects. Journal of Forensic Sciences 37:130±139.
Godown L (1969) Forgeries over genuine signatures.
Journal of Forensic Sciences 14:463±468.
The Threat
Kelly JH (1983) Classification and identification of modern
office copiers. Houston: American Board of Forensic Counterfeited documents are documents that are re-
Document Examiners Inc. produced as originals. The entire document is a fake,
Nickell J (1990) Pen, Ink, & Evidence. Lexington: The e.g. counterfeited banknotes, bonds, checks, vou-
University Press of Kentucky. chers, driving licenses, lottery and admission tickets,
Osborn AS (1929) Questioned Documents, 2nd edn.
stamps, certificates etc. If the reproduction is made by
Albany: Boyd Printing Co.
using materials and procedures which do not corre-
Purtell DJ (1980) Dating a signature. Forensic Science
International 15:243±248. spond to the original, the counterfeit is called an
Starrs J (1991) The case of the doctor who doctored the imitation.
documents. Scientific Sleuthing Review 15:1. Forged documents are altered originals produced
Totty RN (1990) The examination of photocopy docu- by adding, removing or substituting relevant infor-
ments. Forensic Science International 46:121±126. mation or features, for example a modified check
DOCUMENT ANALYSIS/Forgery/Counterfeits 581

amount, an imitated signature on a contract, altered of an original can be produced. This is unfortunately
passport or gift certificate, altered payment card or true not only for low profile documents used in daily
credit card. business, but also for security documents. The full
The counterfeiting of banknotes, bonds, checks or color copier has become the most often used tool for
driving licenses and the illegal reproduction of pass- illegally reproducing value documents. Used as a
ports and travel documents and check and credit card printer device linked with a computer, it offers a
fraud are high profile crimes, committed mostly not wide range of technical features to match the copy
by an individual, but by criminal organizations to the original. Colors can be balanced, numberings
involved in terrorism, illegal cross-border traffic, changed or signatures scanned. Even some of the
drug trafficking, financial transactions or other activ- traditional security features, for example the security
ities of organized crime. By definition, such highly thread of banknotes, can easily be imitated.
valuable so-called security documents, such as bank- Banknotes are documents which are normally not
notes and passports, should be specially protected altered, but counterfeited, however, the situation is
against misuse. The higher the value of a document different in the case of check fraud. Bank checks and
(absolute or relative) and the greater the damage personal checks are value documents which are, or
caused by fraudulent actions, the higher the risk of should be, protected against misuse as banknotes are.
fraud. Therefore, there will be greater protection of However, the level of securities is mostly considerably
the document against fraud. However, a great number lower, making it even easier for criminals to forge.
of these documents are still too easy to counterfeit, Besides reproducing the entire check from scratch, the
because they do not meet the minimum standard of threat of check fraud comes from altered written
manufacturing. If a banknote of reasonable quality checks and forged stolen blank checks. The main
can be reproduced by the use of modern desk top obstacle to altering an already written check is the
publishing or simply with a full color copier, then erasing and substituting of entries without revealing
document securities are missing. If a security docu- the changes. In the case of forged blank checks the
ment, such as a passport, can easily be altered by important challenge for the forger is to imitate the
substitution of the passport owner's photograph or by check owner's signature.
chemical eradication of the passport entries, the docu- Another category of faked documents, similar in
ment securities of the passport do not meet the neces- complexity to check fraud, are counterfeited and
sary standard to prevent counterfeiting and forgery. forged identity documents. The attraction to the crim-
The threat of document counterfeiting and forgery inal forger is ± unlike banknotes and checks ± not the
has become twofold, due to the quantity and the pecuniary value, but the ability to create a new
quality of counterfeited and forged documents. identity. Since modern identity documents, such as
The increase of fake documents of all kinds, the passports or ID cards and driving licenses, are com-
flood of low-profile security document fraud, mainly plex documents with respect to manufacturing and
for immigration and cross-border purposes as well as technical set-up, it is easier and quicker to alter
the frighteningly good quality of some seized cur- genuine documents than to counterfeit an entire docu-
rency counterfeits, give rise to concern, which is much ment. Statistics from immigration services forgery
greater than just a forensic dimension. desk examination make it clear that among the hun-
Document fraud by counterfeiting and forgery is no dreds of different ID documents used worldwide,
longer a form of crime exclusively encountered in there are still too many in which the substitution of
white collar crimes or secret intelligence activities. the identity of the document holder is not a difficult
Document counterfeit and forgery is nowadays task for a skilled forger. As for check fraud, one of the
found in all forms of crime. Statistics regarding the forger's difficulties is the alteration of typewritten
most often counterfeited currency worldwide ± the US and/or handwritten entries. Moreover, the document
dollar ± or the number of seized fake travel documents holder's photograph must be substituted, quite often
at border controls and immigration services, show this is not a difficult task, since the weakest link of
frighteningly high figures for these trans-national most easily altered ID documents is the document's
phenomena. The relevant question of the document protection against substitution of the photograph.
examiner `genuine or false?' is certainly not trivial. Many of these documents have ring staples and dry
What is considered genuine and what is considered or wet stamp cachets to secure the photograph, which
fake? Even valuable documents are too often of such do not give any protection against substitution. Even
poor quality that it is not particularly difficult to thin-laminate sealed photographs are no problem for
imitate them. the professional forger who has little difficulty in
The improved technology of modern office splitting the photograph substrate in half to get access
machines has meant that almost perfect reproductions to the picture without breaking the laminate.
582 DOCUMENT ANALYSIS/Forgery/Counterfeits

A final category of document complexity are the produced by a restricted number of manufacturers
machine-readable documents, such as payment cards and not normally used for the production of common
and credit cards, and also plastic card legitimization documents; for example paper from specialized paper
documents, for example modern identity cards, driv- mills, paper containing special fibers, papers without
ing licenses and the various types of smart cards. The optical brightener, multilayer polymers for synthetic
threat of misuse of plastic money and plastic cards is documents, printing inks of particular spectral com-
inescapable and this form of fraud is increasing position, printing inks specially resistant to chemical
dramatically. Small-scale forgeries involve the alter- eradication or alternatively, especially sensitive to
ing of stolen cards. In addition to altering the entries chemical attacks etc.
on the plastic substrate, the manipulation of the Secondly security printing techniques should be
stored information on the magnetic strip, the electro- used. Besides offset as the most common technique
nic chip, is an additional difficulty. However, this used in printing, special printing techniques such as
obstacle can be overcome if necessary. The threat of letterpress, intaglio (relief printing) and silk-screen
today's card fraud is, however, the amount of orga- are highly effective and not easy to imitate.
nized crime involved in forgery and the counterfeit of Thirdly, additional security attributes should be
plastic cards. The criminal organizations produce used. There are various security devices to improve
blank cards with machine-readable devices, which the protection of a document against counterfeiting
are then delivered to the customer for encoding and, and forgery. These are used either in combination
if necessary, to be completed by printing, embossing with the document materials during manufacturing or
or additional optical security devices (for example added on the printed document. These modern secur-
hologram). ity devices include (among others):
It is true that despite the high quality of some
. Paper securities: watermarks; fluorescent fibers
professionally produced forgeries and counterfeits of
and planchettes;
security documents, the forensic specialists have little
. Printing securities: guilloche pattern (printed back-
difficulty detecting the fraud when checking a ques-
ground pattern of interlacing fine lines); simulta-
tioned document thoroughly. The recipient of these
neous printing (perfect fitting of recto/verso
counterfeited documents, however, is likely to be an
printing); fluorescent and magnetic inks; optically
unskilled and not technically equipped customer.
variable inks (ink taking on different colors
depending on the angle of viewing); rainbow print-
ing (continuous change of colors); microprinting
Document security (extra small printing to be seen only after magnifi-
cation); latent images (visualization depending on
It goes without saying, that there are different classes
angle of viewing); scrambled indicia (scrambled
of documents, according to the probability of fraud,
latent image, only visible through a special lens);
the damage caused by it and the absolute as well as
. Securities of personalized entries: text printed by
the relative value of the document. The class which is
use of matrix printers or laser printer; digitized
best protected against counterfeiting and forgery is
laser printing of text, photos or signatures;
high-value documents, such as banknotes, passports,
. securities to prevent substitution of pages, photos:
checks and credit cards. Whereas other value docu-
staples; embossing stamps (dry stamps), stamp
ments, such as stamps, vouchers, certificates, admis-
cachets; optically variable devices OVD (holo-
sion tickets, stamps, lottery tickets, visa documents,
grams, kinegrams, latent images); protective films
permits and many others may be considered as a class
(thin laminates) with ink printing, retro-reflective-
of lower security value and therefore also of lower
pattern, diffraction grating image; perforations;
document security protection. The understanding of
. machine readable securities: OCR printing; mag-
the counterfeiting and forgery potential of different
netic strips.
types of document demands an enhanced know-how
of the state of the art of technical document securities
and the respective possibilities and limits. Generally
Methods of Examination
speaking, there are three approaches to improve the
technical security of a document against counterfeit- The methodology of document counterfeit and for-
ing and forgery. gery analysis is first a question of where the authen-
First there is the use of exclusive materials and ticity checking takes place. Depending on the form
controlled substances: the higher the value of a docu- of fraud and the type of document involved, a first
ment, the more exclusive the materials for manufac- examination will not necessarily be done in the
turing should be. Controlled substances should be document laboratory, but at the first line of
DOCUMENT ANALYSIS/Forgery/Counterfeits 583

inspection; for example bank institutions, immigra- . Spectroscopic examination of organic compounds
tion services, border controls, social security services (polymers of synthetic documents, e.g. credit cards;
etc. Therefore, the personnel should be capable of identification of photocopy toners).
identifying a false document (in the time available
and technical support) before sending it to the foren- The methodology follows the general system of
sic document laboratory for thorough checking. For forensic examination and standard procedures for
suspect documents, that are not identified by a first document-related examination, based mainly on the
line of checking, the entire examination process will detection of irregularities or by comparing the ques-
be handled by forensic experts to confirm or refute tioned item with an authentic specimen. An absolute
the suspicion. opinion on the authenticity of a questioned document
There are three levels of detection of counterfeited may, however, not always be given, even after high
or forged documents, depending on the authenticity technology examination, simply because the differ-
features to be checked. ences between the original and the suspected imitation
Level 1 includes features, which can be visually are not conclusive. For example, the authentic docu-
examined without special equipment, such as water- ment may be of poor (print) quality, or the highly
marks, security threads, relief structures, latent dangerous forms of fraud and forgery are not known.
images, stamps (dry stamps or stamp cachets), OVD Comprehensive reference manuals and databases
(e.g. kinegrams), mechanical erasure, perforations of for highly protected security documents, particularly
staples, residues of adhesives, obliterated writing and banknotes, passports and visas, try to keep updated
other visible traces of manipulation. collections of images from authentic documents,
Level 2 refers to features to be checked with tech- including close-up views and specifications of rele-
nical facilities such as: visible light to look at printing vant security features. In addition document experts
quality, traces of substitution or relief effects; UV light of immigration and forensic science services are net-
to detect UV-fluorescent securities (paper bleaching, working information on detected counterfeits and
fibers, printing) and UV-visible forgery traces (chemi- forgeries. This is a valuable help for fast authenticity
cal eradication); infrared radiation to examine writing checking, mainly at the front line of inspection. For
inks; reading devices to detect magnetic printing; many other documents however, this type of refer-
special viewing systems for retroreflective securities; ence material does not exist. This is also the case with
reading systems for machine readable devices. documents that had been produced in such a variety
Level 3 examinations have to be carried out in the of `official' types, that it is no longer clear what
document laboratory, with sophisticated equipment should be considered authentic!
that can not be used for field examination. Another factor that is making the unambiguous
identification of suspected questioned documents
. Electrostatic examination of latent impressions of more and more difficult, is the improvement in office
handwriting and detection of indentations (inden- technology; these are also available for use in forgery
tations of erased writings, traced signatures on and other criminal activities. Digital scanners and
checks or passports); printers have reached a technology level able to
. Juxtaposition as well as superposition comparison produce documents, even security documents, of a
with the microscope or by digital image processing, quality standard, which makes it more and more
to visualize faint printing defects; difficult to differentiate between genuine and fake.
. High magnification microscopy examination to This is particularly true for laser color copy technol-
detect trace evidence of mechanical erasure or ogy with its almost unlimited software possibilities.
substitution (residues of writing and printing ink, However, combating color copy fraud is possible and
adhesives; traces of cutting, evidence for manipula- technologies to prevent it are available. Either by
tion of staples, binding, stitching, etc.); programming the scanning software, to recognize
. Infrared absorption and infrared luminescence automatically unauthorized reproduction, e.g. of
examination to decipher nondestructively faint or banknotes and checks, or by printing on the copy a
obliterated writing and printing (traced signatures, latent machine-readable code, which can lead via the
modified stamp cachets); manufacturer to the customer's copy machine.
. Microspectrophotometric (colorimetric) analysis The detection of document counterfeits and forgery
of the spectral composition of colors; is a challenge not only to forensic sciences!
. Chemical detection of chemical erasures;
. Chemical spot reactions to check the organic and See also: Forgery and Fraud: Counterfeit Currency;
inorganic composition of document materials Payment Cards. Document Analysis: Handwriting; Ink
(paper, ink); Analysis.
584 DOCUMENT ANALYSIS/Handwriting

Further Reading known specimen handwriting of the suspected writer


of the questioned material. A document examiner may
ASTM Standard E 1422-98 Standard Guide for Test
be asked to examine such diverse documents as anon-
Methods for Forensic Writing Ink Comparison. West
Conshohocken: The American Society for Testing and ymous letters, writings and/or signatures on cheques,
Materials (ASTM). credit cards and vouchers, wills, mortgages, other
Ellen D (1997) The Scientific Examination of Documents. legal documents, medical records and diaries. It
London: Taylor & Francis. should be noted that, although documents generally
Interpol (ed.) Counterfeits & Forgeries. Amsterdam: consist of writing on paper, the forensic document
Editions Keesing Publishers. examiner can be called upon to examine writings on
Passport Handbook. Amsterdam: Editions Keesing Pub- other less conventional documents such as black-
lishers. boards, whiteboards, pieces of pasta, body parts,
Rapp B (1990) Counterfeit I.D. Made Easy. Port Town- fence posts and graffiti on buildings, walls, windows
send, Washington: Loompanics Unlimited.
etc.
Rapp B (1991) Credit Card Fraud. Port Townsend,
Writing is taught from a model system. The act of
Washington: Loompanics Unlimited.
Rapp B (1991) Check Fraud Investigation. Port Townsend, continuously repeating a written character fixes the
Washington: Loompanics Unlimited. form of that character in the mind of the writer,
Van den Assem B, Brongers D, Rath J et al. (1994) Security normally during childhood, until the production of
Documents ± Practical Guide for the Security Printing this form becomes `automatic'. From the moment
Sector. The Hague: Sdu Publisher. people start learning to write, they introduce devia-
Van Renesse R (ed) (1998) Optical Document Security. tions from the model writing system taught. The
Boston, London: Artech House. extent of these deviations increases as the writing
style becomes more personalized, resulting in a style
which is the product of many factors including the
model system, artistic ability, muscular control, nat-
ure of employment, frequency of writing and expo-
sure to the writings of others. This results in an
individual writing style, the development of which
Handwriting occurs throughout the childhood and adolescent
years, and often beyond (Fig. 1).
M Vos, S Strach and P Westwood, Forensic Document
A similar evolution can be seen in the development
Services, PO Box 543, Queanbeyan, NSW,
of a person's signature with the individual practicing
Australia
a certain style which changes over time under the
Copyright # 2000 Academic Press influence of the factors described above, until an
doi:10.1006/rwfs.2000.0475 often quite individualized pictorial representation of
the person's name is formed.
Document examiners refer to writing characteris-
tics attributable to a model system as being `class' or
Introduction
`style' characteristics. Different styles of writing are
Handwriting can be described as the formation of taught in different countries and sometimes regions,
letters, characters or symbols, using a writing imple- which accounts for some of the variability in hand-
ment, according to a recognizable pattern which is writing styles. The styles taught also change over
designed to communicate with another person. The time. For example, in Australia today it is unusual
issue of who wrote a particular piece of handwriting to be taught the elaborate cursive style of writing
can be central to many legal proceedings. Accepted which was once commonly taught. A number of
ways to prove handwriting include admission by the terms are used by document examiners for characters
writer or the testimony of a person who witnessed the found infrequently in the general population includ-
writing. If the writer is uncooperative or not available, ing `unusual', `personal' or possibly `individual' char-
and there are no apparent witnesses, or the writing is acteristics. The term `individual' characteristic is
otherwise in dispute, then there is a need to resort to somewhat misleading if applied to one letter form,
other means of identification. There are two other since one letter form even if very unusual is not
means acceptable by the courts. The first is to call a sufficient to individualize, or identify, the writer. In
person acquainted with the writer's handwriting; the order to distinguish the writing of any one person, the
second, and more reliable, is to employ the services of handwriting expert builds a background knowledge
a forensic document examiner to undertake a com- over time, from examination of the writings of many
parative examination of the writing in question with different people, of what can be considered common
DOCUMENT ANALYSIS/Handwriting 585

Figure 1 Ten people's writing of the place name `Queanbeyan', illustrating the differences and occasional similarities which arise
in the writings of different people.

or unusual features of handwriting and how hand- tions, alignment of letters and words, slope, speed of
writing can change under different circumstances. If writing, angularity, pressure, shading, diacritics and
sufficient handwriting is present, it is the combination layout are considered.
of rare and common features which serves to identify Constructional details such as line direction and
a particular writer. sequences of separate strokes can often be determined
Although we use the terms forensic document from microscopic examination. Evidence of the
examiner and handwriting expert interchangeably sequence of separate strokes can sometimes be
for the purposes of this article, it should be noted obtained by the direction of any small terminating
that the term `handwriting expert' is sometimes used and commencing strokes between separate parts of a
by those who seek to determine a person's psycholo- character, called spurs or ticks, or faint connections
gical characteristics from their handwriting (graphol- between these parts, showing the direction of pen
ogists). Caution should be taken when engaging the travel just before a pen lift, or just after the pen has
services of a `handwriting expert' to ensure that the contacted the paper following a pen lift. Usually,
person engaged is fully trained and qualified in the examination of several examples of each letter are
field of forensic document examination. Although the required to make such a determination. Ball-point
bulk of the work of most forensic document exam- pen striations can also be used to determine line
iners focuses on the scientific comparative examina- directions as such striations run from the inside to
tion of handwriting and signatures, there are the outside of a curved stroke (Fig. 2). The striation
document examiners who almost exclusively examine pattern present at the conclusion of a stroke, may be
aspects of documents other than handwriting. retained on the ball of the pen and transferred to the
beginning of the next stroke. This `memory effect' can
be used to determine the sequence of strokes. Other
Handwriting Comparison
characteristics of the ball-point pen stroke that allow
When a particular piece of handwriting is called into determination of direction of stroke include `gooping'
question, the handwriting expert will examine the (an extra heavy deposit of ink) following a curve and
writing in question visually, with the aid of hand held dried or reduced ink starts at the beginning of lines.
magnifiers and with a microscope. Features such as For this reason, it is strongly recommended that at
the details of construction of individual letters and least some of any requested set of specimen hand-
letter combinations, comparative height and size writing is written with a ball-point pen. Higher power
relationships of particular letters and letter combina- microscopic (630 or more) examination can also
586 DOCUMENT ANALYSIS/Handwriting

character or its range of variation in the questioned


writing falls outside the range of variation seen in the
specimen handwriting (for example there are differ-
ences in form (shape) or in direction, number or
sequence of strokes), then this is considered to be a
difference. Some differences are regarded as more
significant than others; for example, the repeated
appearance of a different sequence or direction of
strokes for a naturally written block capital letter
(Fig. 3) or numeral may be regarded as a more funda-
mental difference than a shape difference of a lower-
case cursive letter. If significant differences are found,
this usually results in a conclusion that the two writ-
ings are unlikely to have been written by the same
Figure 2 Ball-point pen writings showing (A) striations and person or, alternatively, there is nothing to link the
(B) reduced ink start. Arrows indicate direction of pen questioned and specimen writings as being by one
movement. person. Although definite negative conclusions are
sometimes justified, they should be used sparingly
and in special circumstances. This is because of the
sometimes be used to determine line direction of ball- possibility which should be considered, although
point pen writing, and almost always for pencil writ- remote, of a person having two distinct handwriting
ing. This can be determined from the build up of ink styles. With similarities also, some are more signifi-
or graphite predominantly on the `upstream' side of cant than others. The significance is greater if it
the paper fibers which faced the approaching writing involves a handwriting feature found more rarely in
tip. the appropriate general population. This is usually
The known writing of the person suspected of assessed subjectively, based on the experience of the
writing the questioned document is examined in a examiner in observing handwriting features in very
manner similar to that employed with respect to the many writers. The keeping of handwriting reference
questioned document. Specimen handwriting needs collections, and using such references to determine the
to be comparable in style with the questioned hand- rarity or otherwise of a particular feature, is becoming
writing. For example, questioned upper case block more prevalent and can provide some statistical back-
printing should be compared with specimen upper ing for the results of handwriting comparisons.
case block printing. It is also preferable to examine Finally the examiner, before arriving at a finding,
specimen writing on forms of a similar printed layout. assesses the results of the comparison process in terms
Once suitable specimen handwriting is obtained, an of similarities and/or differences in all comparable
initial comparison is made between the known writ- features of the handwriting and their significance. If
ing samples to ascertain if they can reasonably be no significant differences are found, the examiner
taken to have been written by the one person; it can assesses the likelihood that all of the similarities
happen that documents which are said to bear the could have occurred by chance in the writings of
known writing of one person may include (at times two people, or alternatively could have originated in
unwittingly) the writing of other persons. This is a one person simulating the handwriting of another
particular consideration when using diaries, address without leaving any evidence (see below) of the simu-
books or other documents where many persons may lation process. These are usually assessed as subjective
have had access to and written on the document. probabilities, based on the premise (from the prob-
From the examination of specimen writing, the expert ability multiplication law for independent events) that
assesses how the writing of this particular individual the probability of very many similar handwriting
varies within itself. features, not necessarily individually rare, occurring
Once the requirements for suitable specimen hand- in combination in the writings of two people is
writing are met, the questioned and specimen hand- considered extremely small or negligible. Where the
writings are compared with each other, and an probability of a chance match in the writings of two
assessment made of similarities and differences people and the probability that another person has
between the two writings. A character in the ques- successfully simulated the writing style of the writer of
tioned writing is considered to be similar if it, or its the specimens are both considered negligibly small,
range of variation, falls within the range of variation the document examiner reaches an unqualified con-
for this character in the specimen handwriting. If the clusion that the writer of the specimens wrote the
588 DOCUMENT ANALYSIS/Handwriting

hood of a chance `match' occurring between an idio- Valentine's Day card to a note demanding money
syncratic signature and the handwriting of another from a bank teller.
person writing this signature style without knowledge Obvious features of a person's handwriting, such as
of the genuine signature is significantly reduced. slope and/or size, are often changed as a form of
For most, but not all signature cases, which involve disguise. Some people will deliberately introduce let-
the comparison of a set of specimen signatures with ter forms that are markedly different from their usual
generally pictorially similar questioned signature(s), letter forms and some will use the unaccustomed hand
the main issue is whether the questioned signature was in their writing. In the case of the latter, the writing
written by the writer of the specimens, or whether the produced can display the effects of poor pen control
signature was written as a simulation of the style of and appear untidy, but the subconsciously produced
the genuine signature. It is not possible to determine letter constructions and proportions may remain
by handwriting/signature comparison methods who approximately the same, with the possible exception
wrote a completely simulated signature (or handwrit- of direction of some strokes. The major difficulty with
ing). In such a simulation the normal handwriting most disguised writing is maintaining the disguise
features are distorted by the simulation process. In less over a length of text. Where a basic disguise may
complete simulations there may be evidence of the succeed over a few words, the disguise is often for-
writer's natural handwriting characteristics. For most gotten over a number of sentences with the writer
handwriting comparison cases, the main issue for reverting to a natural handwriting style.
generally similar writings is whether the writer of A totally different style of writing may also be used
the specimens wrote the questioned writing, or as a disguise. For example, if a person normally uses
whether a chance match has occurred in the writing printed script, cursive writing may be tried as a
of two people. It should be stressed that this does not disguise. In this instance however, there will often
mean that handwriting and signature comparisons are be examples of the alternative style available for
fundamentally different, simply that there is often a comparison. There are, nevertheless, some forms of
different emphasis. carefully disguised writing for which determination
of the writer from handwriting comparison methods
General Considerations for Handwriting may be difficult or impossible.
and Signature Comparisons Disguised signatures are written by those persons
intending to later deny the signature they have writ-
For all handwriting and signature comparison cases,
ten. Generally the signature produced is so close to the
three main hypotheses to explain the observations
specimen signatures, except for one or two differ-
must be considered.
ences, that the document examiner will identify the
1. The writer of the specimens wrote the questioned signature as being genuine despite the attempted dis-
material. guise (Figs 5 and 6). Self-simulation of a signature or
2. A person other than the writer of the specimens handwriting as a form of disguise can be significantly
wrote the questioned material, any similarities to more difficult or impossible to detect. The writer of a
the questioned writing having arisen by chance completely simulated signature or piece of writing
coincidence. may be impossible to determine.
3. A person other than the writer of the specimens
wrote the questioned material, any similarities to
Simulation of Handwriting and Signatures
the questioned material having arisen because of a
simulation process. Simulation of handwriting presents a different set of
problems for the potential offender. The person in-
Complications which may further arise are considera-
tending to copy the writing of another person needs to
tions of distortion, disguise or self-simulation of the
obtain some specimen writing of the other person.
writing in the case of hypothesis (1), combinations of
The copyist then has a number of options. The hand-
possible effects of (2) and (3), multiple writers etc.
writing can be `drawn' or some form of tracing can be
The document examiner needs to consider all feasible
used to produce the simulated writing. When drawing
possibilities which might explain the observations
the writing, the copyist must stop frequently to check
and be aware of the danger of not considering all of
the construction of letters and words used by the
these possibilities.
person whose writing is being simulated, considerably
reducing the speed of the writing process. Where no
Disguise of Handwriting and Signatures
examples of a particular letter are available, the copy-
There are many types of documents on which dis- ist must use some other form. As a result, the writing
guised handwriting appears, from an innocently sent thus produced is generally slowly written; the words
DOCUMENT ANALYSIS/Handwriting 589

A
C

Figure 5 (A) Genuine signature `C. Lewis' and (B) slowly written freehand simulation of (A); (C) detail of the letter `e' from the
simulated signature exhibiting poor line quality; (D) detail of the letter `w' from the simulated signature showing uncharacteristic
pen lift.

Figure 6 (A) Genuine signature `C. Lewis' and (B) quickly written freehand simulation of (A). Signature (B) shows errors in letter
proportion and size relationships and an uncharacteristic pen lift in the final part of the signature (C).

may display many pauses in writing where none writing, it will almost certainly be slowly completed
would usually occur, and the letter forms may display with many stops and pauses in the ink line. Tracing
many more variations than the genuine writing since methods can also leave evidence on the questioned
the desire to copy the correct letter conflicts with the document, depending on the method used. For exam-
copyist's natural way of writing a particular letter. ple, traced guide lines in the form of pencil transfers,
There may be evidence of the copyist's natural hand- carbon copy writing and indented impressions can
writing style in the simulation and other differences also usually be detected by the techniques available to
from the genuine writing, especially where an attempt the forensic document examiner. Should the specimen
is made to write fluently. document from which the simulation has been made
Tracing is the second major method of simulating become available, this may also bear such evidence in
handwriting, with the appropriate letters and words terms of matching of particular letter or word forms,
drawn from a pool of writing available to the forger. or indentations of the questioned writing.
This method, although it can be effective in suppres- Signatures contain many fewer letter forms than
sing the handwriting style of the copyist, almost handwriting but present the same problems to the
inevitably still has the problem of lack of fluency in person attempting the simulation. A further difficulty
the writing. Even if at first glance the writing pro- arises with elaborate signatures as it can be difficult to
duced appears to be pictorially similar to the genuine determine directions and sequence of strokes. As with
590 DOCUMENT ANALYSIS/Handwriting

handwriting, it is extremely difficult to maintain both Examination from Reproduction


fluency and the correct forms of the signature com- Documents
ponents. Where the copyist attempts a highly fluent
simulation, evidence of the simulation process is Reproductions of handwritten documents, in the form
usually present in the form of substantial departures of photocopies, fax copies and computer-imaged
from normal letter constructions. In the case of many reproductions are often submitted to the document
traced or freehand simulated signatures, as only one examiner either as questioned or specimen writings.
or two signatures are often used as models, the Of necessity, examination from a reproduction pro-
signatures may be far more similar to each other vides reduced information. For example, microscopic
than would be expected of a group of genuine signa- examination is of little value, except in determining
tures. This is especially true for tracings. the nature of the copy, since the fine details of writings
are lost in the resolution provided by the copy. Never-
theless, useful handwriting comparisons can be made
from examination of reproduced documents, particu-
Factors Influencing Handwriting larly good clear modern photocopies, although the
Many factors can affect a person's handwriting, and examination is restricted mainly to the grosser pictor-
no one person writes so consistently that each letter ial features of the writing. Suitably qualified conclu-
form is exactly the same. However, the relative sions are usually expressed, along with warnings that
method of construction, letter proportions etc. it cannot necessarily be assumed that a true reproduc-
remain consistent within a small range of variation, tion of an original document has been examined as
even if the writing is completed on an uneven surface, photocopy or computer manipulation may sometimes
at speed or under some other stress. More significant be accomplished without leaving evidence of the
variations in writing style are caused by such factors manipulation in the resulting reproduction.
as age, injuries, illness (mental or physical) with
handwriting showing a reduced speed, tremor in the Other Examinations
form of erratic impulse movements and there may
also be misplaced or poorly joined strokes. Attempted Document examination encompasses much more
imitation of this type of writing sometimes shows a than comparison of handwriting and signatures.
high frequency tremor which may not be consistent Document examiners also consider the inks and print-
with the tremor of the genuine writer, inconsistent ing seen on the paper, the paper itself, folding, stamps
letter constructions or careful retouching of strokes and seals and writing impressions. All of these,
which exceeds the skill of the genuine writer. At times together with the handwriting and signatures, go
the use of medication can improve a person's hand- towards proving the provenance of any particular
writing for a limited period of time, and this is questioned document.
considered by the document examiner.
See also: Document Analysis: Analytical Methods;
The only scientific method of determining whether
Forgery/Counterfeits; Ink Analysis; Document Dating.
a piece of handwriting or a signature has been written
by a particular person whose handwriting may have
been affected by such factors as described above, is to Further Reading
obtain as much comparable writing as possible, writ- Conway JVP (1959) Evidential Documents. Springfield, IL:
ten when the factors applied. This normally means Charles C Thomas.
obtaining handwriting or signature specimens written Ellen D (1997) The Scientific Examination of Documents,
as close as possible to the date of the questioned 2nd edn. London: Taylor & Francis.
material. Harrison WR (1966) Suspect Documents. 2nd impression
Determination of the genuineness or otherwise of with supplement. London: Sweet & Maxwell.
handwriting and/or signatures of the infirm can be Hilton O (1982) Scientific Examination of Questioned
among the most difficult examinations which the Documents, revised edn. New York: Elsevier Science.
forensic document examiner undertakes, especially Haber RA and Headrick AM (1959) Handwriting Identi-
fication: Facts and Fundamentals. Boca Rota, FL: CRC
when adequate specimen signatures or handwriting
Press LLC.
reflecting the infirmity are lacking. Nevertheless, use-
Osborn AS (1922) The Problem of Proof. Albany, NY:
ful, if not always certain, determinations can be made Matthew Bender.
by the standard methods of careful examination and Osborn AS (1929) Questioned Documents, 2nd edn.
comparison with the available specimens, looking in Albany, NY: Boyd Printing.
particular for similarities or differences in the more Osborn AS and Osborn AD (1946) Questioned Document
subtle features of the handwriting and/or signature. Problems, 2nd edn. Albany, NY: Boyd Printing.
DOCUMENT ANALYSIS/Ink Analysis 591

Ink Analysis tion date for each type of ink or certain ingredients in
the inks is useful for dating inks.
R L Brunelle, Brunelle Forensic Laboratories,
Fredericksburg, VA, USA
Copyright # 2000 Academic Press Carbon (India) ink

doi:10.1006/rwfs.2000.0479 In its simplist form carbon inks consist of amorphous


carbon shaped into a solid cake with glue. It is made
into a liquid for writing by grinding the cake and
suspending the particles in a water±glue medium. A
Introduction
pigmented dye may be used to improve the color.
Chemical and physical analysis of inks on questioned Liquid carbon inks are also commercially available. In
documents provides valuable information regarding the liquid carbon inks shellac and borax are used in
their authenticity. Comparison of these chemical and place of animal glue and a wetting agent is added to
physical properties of two or more inks can deter- aid in the mixing of the shellac and carbon. Carbon
mine: (1) if the inks were made by the same manu- inks are insoluble in water, very stable and are not
facturer; (2) in some cases, whether the inks are decomposed by air, light, heat, moisture or microbio-
products of the same production batch; and (3) the logical organisms. This class of ink has been available
first production date of the specific ink formulation for more than 2000 years.
involved. When dating tags are detected, it is possible
to determine the actual year or years when the ink
was manufactured. Dating tags are unique chemicals Fountain pen inks
that have been added to ball-point inks by some ink
There are two types of fountain pen inks: (1) iron-
companies as a way to determine the year the ink was
gallotannate type and (2) aqueous solutions of syn-
made.
thetic dyes. Modern inks of type (2) contain synthetic
Relative age comparison tests performed on inks of
blue dyes to provide an immediate blue color to the
the same formula and written on the same type of
ink which gradually turns black after oxidation on
paper with the same storage conditions (performed by
paper. This explains the origin of the name blue±
measuring changing solubility properties of inks) can
black fountain pen ink. This class of ink is also very
estimate how long inks have been written on paper.
stable. This ink is insoluble in water and cannot be
This is done by: (1) comparing the rates and extents of
effectively erased by abrasion. The most popular
extraction of questioned and known dated inks in
fountain pen ink (developed in the 1950s) consists
organic solvents by thin-layer chromatography (TLC)
of an aqueous solution of synthetic dyes. These inks
densitometry; (2) comparing changes in dye concen-
are bright and attractive in color, but they are not
trations by TLC and TLC densitometry; and (3)
nearly as stable as the carbon or blue±black inks.
comparing the volatile ink components by gas chro-
Some of the synthetic dyes used fade and are soluble
matography±mass spectrometry (GC-MS). In cases
in water. The most modern inks of this type contain
where known dated writings are not available for
pigmented dyes, such as copper phthalocyanine
comparison with questioned inks, accelerated aging
(introduced in about 1953) which makes these inks
(heating the ink to induce aging of the ink) can
much more permanent.
sometimes be used to estimate the age of ink using
any or all of the above described techniques. Iron-
based inks can be dated by measuring the migration Ballpoint inks
of iron along the fibers of the paper by Scanning auger
microscopy. The ballpoint pen was developed in Europe about
This article describes state of the art procedures for 1939 and was initially distributed in Argentina about
the chemical and physical comparison, identification 1943. In 1946, several million Reynolds ballpoint
and dating of inks on questioned documents. pens reached the market in the United States.
Ballpoint inks consist of synthetic dyes (sometimes
carbon or graphite is also added for permanence) in
various glycol solvents or benzyl alcohol. The dyes in
ballpoint inks can consist of up to 50% of the total
Composition of Major Types of Writing Inks
formulation. Several other ingredients are usually
Knowledge of the composition of inks is necessary to added to the ink to impart specific characteristics.
understand the reasons for the various methods used These ingredients consist of fatty acids, resins, sur-
to analyze inks. Also, knowledge of the first produc- face active agents, corrosion control ingredients and
592 DOCUMENT ANALYSIS/Ink Analysis

viscosity adjustors. The fatty acids (oleic is the most additives similar to those in rolling ball marker inks
common) act as lubricants to the ball of the pen and and fountain pen inks. The water-based inks are
they also help the starting characteristics of the ball obviously water soluble, whereas the xylene-based
point. inks are water resistant and can only be dissolved
Ballpoint inks made before about 1950 used oil- with strong organic solvents. Formamide or glycol
based solvents such as mineral oil, linseed oil, reci- solvents are essential ingredients in fiber tip inks to
noleic acid, methyl and ethyl esters of recinoleic acid, keep the fiber tip from drying out. Fiber tip inks that
glycerin monoricinoleate, coconut fatty acids, sorbi- contain metalized dyes are light fast.
tal derivatives, and plasticizers such as tricresylpho-
sphate. Modern ballpoint inks (post-1950) are
referred to as glycol-based inks, because of the com- Gel-pen inks
mon use of ethylene glycol or glycol derivatives as a The most recent development in the writing instru-
solvent for the dyes. Benzyl alcohol is also commonly ment industry is the introduction of the gel-pen by the
used as the vehicle (solvent) by some ink manufac- Japanese. Four brands of gel-pen inks have been
turers. Chelated dyes (introduced commercially introduced: (1) the Uniball Signo by Mitsubishi; (2)
around 1953) are stable to light. Red, green, yellow the Zebra J-5; (3) the Pentel Hybrid; and (4) the
and other colored chelated dyes are now used for Sakura Gelly Roll pen. These pens have been mar-
various colored ballpoint inks. keted by the Japanese since the mid-1980s and a
Pressurized ballpoint inks were developed about limited supply of the pens was sold in the United
1968. These pens contain a pressurized feed system States about 1993. Two US manufacturers are now
instead of gravity flow. The physical characteristics of producing these pens.
these inks are quite different from the standard glycol Gel inks contain completely insoluble colored pig-
based ballpoint inks. The composition is basically the ments rather than organic dyes. Writing with this ink
same, but this ink does not become fluid until dis- is very similar to the appearance of the writing with a
turbed by the rotation of the ball point in the socket. ballpoint pen. This ink, which is water based, is a gel
Cartridges containing this ink are under the pressure and not a liquid. It is insoluble both in water and
of nitrogen or some other inert gas. The positive strong organic solvents. This physical property makes
pressure on the ink allows the pen to write in all it impossible to analyze (by traditional methods) for
positions and in a vacuum. These pens are used by the purpose of comparing two or more inks of this
astronauts during space travel. type.

Rolling ball marker inks

Rolling ball marker inks were introduced in Japan in Ink Comparisons and Identifications
about 1968 and shortly thereafter in the United
Inks are usually examined for three reasons:
States. These inks are water based and usually contain
organic liquids such as glycols and formamide to 1. To compare two or more ink entries to determine
retard the drying of the ball point. The dyes in these similarities or differences in inks which can pro-
inks are water soluble or acidic dye salts. The light vide information concerning whether entries have
fastness of these dyes range from good for the metal- been added or altered.
ized acid dyes to poor for some of the basic dye salts. 2. To determine if two or more entries were written
Water fastness is usually poor, except that some of with the same formula and batch of ink, thus
these dyes have an affinity for cellulose fibers in paper providing a lead as to whether certain entries
which produces a degree of water fastness. Water- could have been written with the same pen.
resistant rolling ball marker inks are also available. 3. To date ink entries to determine whether docu-
These inks are totally insoluble in water and can only ments have been back-dated. This section deals
be dissolved in strong organic solvents, such as pyr- with the first two reasons for analyzing inks.
idine or dimethylsulfoxide (DMSO).
Nondestructive methods of comparison should be
carried out first, because chemical analysis causes
minor damage to the document by removing ink
Fiber or porous tip pen inks
samples for analysis. Typically, the nondestructive
This class of inks was developed in Japan about 1962 methods include: (1) a visual and microscopic exam-
and in the United States about 1965. Fiber tip inks are ination of the writing to assess its color and the type
usually water or xylene based and contain dyes and of pen used; (2) infrared reflectance and luminescence
DOCUMENT ANALYSIS/Ink Analysis 593

examinations to determine whether the inks reflect or . Note and record the color of the ink in solution and
absorb infrared light and whether the inks luminesce; then spot the ink on to the TLC plate using the
and (3) viewing the inks under long- and shortwave 10 ml micropipette. Keep the spots small by spot-
ultraviolet light to determine if the inks are fluores- ting intermittently and allowing the spots to dry
cent under these wavelengths of light. Often these between each spotting.
techniques are sufficient to determine if two or more . Repeat the above for all ink samples to be com-
inks are different. However, if these techniques fail to pared. Up to about 20 samples can be spotted on the
detect any differences in the inks, then further che- same TLC plate. Be sure to analyze a sample of the
mical analysis is necessary to determine if the inks paper with no ink as a control.
being compared really have the same formula. . Place the TLC plate with the spotted inks in a
The most widely used technique for comparing and temperature-controlled oven for approximately
identifying inks is TLC. This technique separates the 10 min at 808C. Allow the plate to cool to room
dyes in the ink and the invisible organic components temperature then place the plate in the developing
in the ink. This allows a direct comparison of the chamber using a solvent system of ethyl ace-
composition of inks being examined on the same TLC tate:ethanol:water (70:35:30 by vol.). The solvent
plate. To determine the relative concentrations of system should be allowed to equilibrate in the
dyes present in the ink, the dyes separated on the developing chamber for at least 15 min.
TLC plate are scanned in a TLC scanning densit- . Allow the TLC to develop for 15 min, then remove
ometer. The method is fast, reliable and inexpensive. it from the chamber and dry in the oven for
High performance liquid chromatography (HPLC) approximately 15 min at 808C.
has also been used for comparing inks with some . View the developed TLC visually and under ultra-
success. Gas chromatography-mass spectrometry violet light to determine which inks match in
(GC-MS) is a very useful technique but the equipment terms of the dyes and fluorescent components
is expensive. present.
. Scan the plate in the scanning TLC densitometer to
measure the relative concentrations of the dyes
Method of chemical analysis present in the inks. The dyes are scanned at
Equipment, materials and solvents 585 nm for blue and black inks if a spectrometer
type densitometer is used. Video densitometers see
. Merck HPTLC plates (silica gel without fluores- all spots in shades of black and therefore no wave-
cent indicator). The plates should be activated at length setting is needed for this instrument. (If the
1008C for 15 min before use. above solvent system did not adequately separate
. TLC scanning densitometer the dyes in the ink for accurate densitometer read-
. Reagent grade pyridine, ethyl acetate, 1-butanol, ings, repeat the tests using 1-butanol:ethanol:water
ethanol, benzyl alcohol, DMSO, and water (50:10:15, by vol.).
. 1 dram (1.8 g) glass vials with screw caps . Compare the relative concentrations of the dyes
. 10 ml and 4 ml disposable micropipettes present in the various inks. Failure at this point to
. TLC glass developing chamber to accommodate detect any significant differences among the inks
standard 4 in 6 8 in (10 cm 6 20 cm) TLC plates compared justifies a conclusion that all inks are
with cover consistent with being of the same formulation. This
. 20 guage syringe needle and plunger (the point of statement is based on the finding of no significant
the needle must be filed so that the point is flat) differences in the nondestructive tests and the che-
. 10 ml and 20 ml automatic pipettes mical analysis. It should be noted that complete
. temperature controlled oven identification of an ink is not possible, because not
all of the original ingredients in ink are present in
ink dried on paper.
Procedure
. To identify the manufacturer and specific formula-
. Using the syringe needle and plunger, punch out tion of questioned inks, standard inks of known
about 10 plugs of ink from the written line. manufacture and formulation must be analyzed
. Place the plugs in the glass vial and add 1±2 drops simultaneous with the questioned inks using the
of the appropriate solvent to the vial to dissolve the same procedures described above. To do this, how-
ink (usually pyridine for ballpoint ink and ethanol ever, requires access to a complete and comprehen-
and water (1:1) for nonballpoint inks. Water resis- sive collection of standard inks and an analytical
tant nonballpoint inks may require using pyridine method that distinguishes each standard. The
or DMSO). Allow 15 min for the ink to dissolve. strength of any identification is only as strong as
594 DOCUMENT ANALYSIS/Ink Analysis

the completeness of the standard ink reference by the Formulab Company since before 1970; how-
collection and the ability to identify its inks. ever, the use of tags in their inks was discontinued in
. If the ink is properly identified, it is possible to June 1994. Since the tags are considered proprietary
determine from the manufacturer when that speci- information by Formulab, no further information
fic formulation of ink was first made. This may about the tags can be reported here. Formulab
determine if a document was backdated. should be contacted directly, if this information is
needed.
Although the above procedures are the most com- Ink dating tags are detected and identified by TLC
monly used and have withstood the test of the courts using a solvent system of chlorobenzene and ethyl
for the comparison and identification of inks, other acetate (5:1, v/v). Standard samples of the tags should
methods are available. For example, gas chromato- be run simultaneously on the same TLC plate as the
graphy (GC) and GC-MS can be used to detect any questioned inks. The tags, if present, are viewed
volatile organic ingredients that might be present in under longwave ultraviolet light and the RF values
the inks. HPLC can be used to detect volatile and of the tags present in questioned inks are compared
nonvolatile components. Electron microscopy can be with the RF values of the standard tags. The dates
used to distinguish carbon from graphite, when these the various tags were used must be obtained from
are present in inks. Time and the amount of ink Formulab.
sample available for analysis usually make the use
of these techniques impractical.
Relative age comparison methods

Dating inks by this procedure is based on the scien-


Dating of Inks tifically proven premise that as ink ages on paper,
there are corresponding changes in the solubility
As mentioned earlier in this article, there is a huge
properties of the inks. Therefore, by comparing the
demand for the dating of inks on questioned docu-
solubility or extraction properties of questioned inks
ments. Any time during an investigation when there is
with known dated inks of the same formula on the
some question about the date of preparation of a
same type of paper and stored under the same condi-
document, an ink dating chemist is needed. Over the
tions, it becomes possible to estimate how long the
past 30 years, the ability to perform these examina-
ink has been written on the document. Two or more
tions has become widely known and recognized
inks of the same formulation can be compared with-
among forensic scientists, document examiners and
out known dated writings to determine whether the
attorneys throughout the world. The ink dating pro-
writings were made at the same or different times.
cedures that will be described have passed the Frye
This is only true if the inks being compared are still
and Daubert tests on numerous occasions and are
aging (drying), because after the ink has aged out
therefore routinely accepted in US courts. Testimony
(completely dry), no differences in solubility proper-
has also been admitted using these techniques in Israel
ties are expected, even if the inks were written at
and Australia.
different times. Typically inks will become totally dry
(as measured by these procedures) within 6 years;
First date of production method some inks become dry in less than 6 years.
When two or more matching inks are compared
After the ink is uniquely/positively identified, the first without known dated writings, it is still possible to
date of production of that ink or certain ingredients in determine the sequence in which the inks were writ-
the ink is determined from the manufacturer of that ten. This again requires knowing that the inks are still
specific ink formulation. If the ink was not made until aging and also knowing how the inks age. For exam-
after the date of the document, then it can be con- ple, some inks extract faster and more completely in
cluded that the document was backdated. If the ink organic solvents as the ink ages; whereas, others
was available on the date of the document, then the extract more slowly and less completely as they age.
document could have been written on that date. To determine which way the ink ages, a sample of the
ink is heated at 1008C for 30 min. The rate and extent
of extraction of this heated sample into an organic
Ink tag method
solvent is compared with an unheated sample of the
If an ink tag is identified in an ink, it is possible to same ink to determine if the heated (totally aged)
determine the actual year or years when an ink was sample extracted faster and more completely than the
made. Tags have been added to some ballpoint inks unheated sample, or vice versa.
DOCUMENT ANALYSIS/Ink Analysis 595

R-Ratio (rate of extraction) method and percent . Calculate the various R-ratios for each sample by
(extent) extraction method letting the percent of ink extracted in the weak
solvent at 10 min equal one. Then calculate the R-
. Using the syringe and plunger, remove 10±15 plugs
ratios for each time interval of 0.5, 1.5, 3 and
of ink and paper and place them into 1 dram glass
10 min. This gives a normalized curve.
vials. Cap and label the vial with the sample
. To obtain R-ratio curves, plot R-ratios vs. time of
number. Repeat for each sample to be analyzed.
extraction (Fig. 1). Since all samples are being
. Set the timer to 10 min.
compared in the same manner, it is not necessary
. Using the automatic 20 ml pipette, add 20 ml of a
to correct for volume changes caused by successive
weak solvent to the vial containing the ink sample
aliquots removed from the vials.
and start the timer immediately. (For almost all
. Compare the R-ratio curves of all inks tested of
ballpoint inks, 1-butanol is a good weak solvent.)
the same formulation. To estimate the age of the
. Stir by rotating the vial containing the ink and
questioned inks, compare the R-ratio curves of the
weak solvent immediately after adding the weak
questioned inks with known dated inks.
solvent and just before each aliquot is removed for
. Calculate the percentage or extent of ink extracted
spotting.
in the weak solvent at the various time intervals, by
. Spot 4 ml aliquots of dissolved ink in one contin-
dividing the reading for each weak solvent spot by
uous application on a TLC plate at 0.5, 1.5, 3 and
the total amount of ink extracted in the weak and
10 min intervals. Place these spots side by side at
strong solvent, then multiply by 100. Figure 2
one end of the plate approximately 1 cm apart. (It
shows the amount of ink extracted in 10 min.
may be necessary to use tweezers to remove the
pipette from the vial.) Note: If a nonballpoint ink is A simplified percent extraction procedure can be
being analyzed, it may be necessary to spot the 4 ml performed by extracting each sample for just 1 min
aliquots intermittently to prevent the spot from in the weak solvent, then after spotting this 1 min
getting too large. The spot should be no larger extract, the strong solvent is added directly to the
than 0.3 cm in diameter weak solvent remaining in the vial. Then after allow-
. Repeat the above procedures for each sample to be ing the strong solvent to extract for 15 min a second
analyzed. aliquot is spotted. This procedure produces just two
. Evaporate the solvent remaining in the vials in an spots to measure in the densitometer. Although R-
oven at 808C (about 15 min). ratios cannot be determined by this procedure, accu-
. Remove the vials from the oven and allow them to racy and reproducibility of the percent extraction
cool to room temperature.
. Using the automatic pipette, add 10 ml of a strong
solvent to each vial and allow to extract for 15 min.
(Benzyl alcohol is the solvent of choice for ball-
point inks and some nonballpoint inks. Some non-
ballpoint inks may require using ethanol:water
(1:1) or DMSO for water resistant nonballpoint
inks.)
. Spot 4 ml of the ink extracted with the strong
solvent adjacent to the weak solvent spots. (If
benzyl alcohol is used for the strong solvent, spot
in one continuous application of the pipette to the
plate. If pyridine is used, spot intermittently to keep
the spot from getting too large.)
. Repeat the above steps for each sample.
. Dry the spots on the TLC plate at 808C for about
15 min.
. Remove the plate from the oven and allow to cool
to room temperature.
. Scan the plate in the scanning TLC densitometer
along the path of the four weak solvent spots and Figure 1 R-Ratio curves (rates of extraction) for Bic black
the one strong solvent spot and read the relative ballpoint inks written in 1992, 1994, 1996 and 1998. The match-
concentrations of the five spots. ing curves for the 1992 and 1994 inks means that the ink
. Repeat the scan described above for each sample. became totally dry after 4 years, because there was no change
in rate of extraction after this time.

Você também pode gostar